LSAT and Law School Admissions Forum

Get expert LSAT preparation and law school admissions advice from PowerScore Test Preparation.

User avatar
 Dave Killoran
PowerScore Staff
  • PowerScore Staff
  • Posts: 5853
  • Joined: Mar 25, 2011
|
#85538
Complete Question Explanation
(The complete setup for this game can be found here: lsat/viewtopic.php?f=307&t=9348)

The correct answer choice is (B)

The question asks for a pair of buildings that can be inspected together only on Monday. G is a natural building choice for analysis since G is so restricted: G cannot be inspected on Wednesday, and if G is inspected on Tuesday, it would have to be inspected on Tuesday morning, forcing J to be inspected on Tuesday afternoon. Thus, on Tuesday, G must be inspected with J. Of course, at this point, you may be asking, “Why are we talking about Tuesday? Isn’t this question about Monday?” Yes, it is, but consider G’s situation at this juncture: it cannot be inspected on Wednesday, and on Tuesday it must be paired with J. Thus, if G is to be paired with any other variable besides J, that pairing must occur on Monday. Therefore, any answer choice pairing G and L, V, or Z would be correct. Only answer choice (B) makes such a pairing.

Note that the hypothetical created in question #13 answer choice (D) can be used to eliminate answer choice (A) from contention.
 raye
  • Posts: 4
  • Joined: Aug 27, 2013
|
#10592
I've seen some people ask about this same question, but in their version of the test it sounds like it's not a logic game...I printed the version on the take-home practice test section of the website.

I'm trying to understand why the answer is (B) and not (D). I chose answer choice (D) because I thought it forced the rest of the variables into a single slot, whereas answer choice (B) two of the variables can be interchanged. Obviously I did something wrong, but I'm not sure what it is I'm missing.
 Jon Denning
PowerScore Staff
  • PowerScore Staff
  • Posts: 904
  • Joined: Apr 11, 2011
|
#10596
Hi raye - thanks for the question. It may simply depend on where people got their copy of the test (or even how they chose to represent it; for instance, "Logical Reasoning Section 2" may simply indicate the second LR section, rather than section 2 of the test).

Anyway, if you're asking about game question #15, I think I can help you out:

For answer choice (D), inspecting L and S on the same day could happen on Tuesday, and thus is not restricted to just Monday. Here is a potential way for LS to be on Tuesday (morning variable on bottom, afternoon variable on top):

Tuesday:

J L Z
G S V

So clearly LS together is not limited to just Monday.

For answer choice (B), however, if we inspect G and V together, they must be inspected on Monday. We cannot have G and V together on Wednesday since G is a hotel and hotels are not inspected on Wednesday. And we cannot have G and V together on Tuesday because that would put J on Wednesday (after G), and J is a hotel so it cannot go Wednesday.

Here is what G and V on Monday could look like:

V J S
G L Z

So because G V can go Monday, but nowhere else, that is the correct answer.

Thanks!
 raye
  • Posts: 4
  • Joined: Aug 27, 2013
|
#10597
OH. So I completely misunderstood the question. I thought the question was asking if certain variables were placed on Monday, which one would force all the other variables to take specific slots. I hadn't even thought about what if those same variables were placed on a different day.

Thank you very much for explaining it to me!
User avatar
 Adam354
  • Posts: 29
  • Joined: Feb 08, 2022
|
#93680
Since G is confined to the first three spots, it seems logical to inspect the two answers containing G first.
If you wrote down the question number 1 sequence: LSGJZV, you can eliminate answer A.

For the second possibility containing G: Since GV can only fit on two days, Monday or Tuesday, all we have to do is test whether it can work on Tuesday.

It does not, so B is the correct answer, since GV is forced to be together on Monday.
 Adam Tyson
PowerScore Staff
  • PowerScore Staff
  • Posts: 5153
  • Joined: Apr 14, 2011
|
#93690
Correct, and that is exactly how I went about it, Adam! You could have also focused on J, though, because it is also confined to just three spaces and appears in two answer choices. That would have eliminated answers A and C, at which point the sensible thing would again have been to do as you did and test answer B.

Get the most out of your LSAT Prep Plus subscription.

Analyze and track your performance with our Testing and Analytics Package.